LSAT and Law School Admissions Forum

Get expert LSAT preparation and law school admissions advice from PowerScore Test Preparation.

 Robert Carroll
PowerScore Staff
  • PowerScore Staff
  • Posts: 1787
  • Joined: Dec 06, 2013
|
#74137
tfab,

My view of answer choice (E) is that it operates by presenting a general rule which the stimulus situation may exemplify. The answer tells us to expect more aggressive behavior after someone is told it's acceptable. The children in the stimulus behaved more aggressively after playing games. Is that because they were told it was acceptable, or otherwise viewed the experiment as entailing that it was acceptable? If so, their behavior would make sense as a consequence of that. This doesn't prove anything definitively, but it provides an explanation consistent with the facts and conclusion, so it bolsters the conclusion.

Robert Carroll
User avatar
 SGD2021
  • Posts: 72
  • Joined: Nov 01, 2021
|
#94328
Hello, would it be correct to say that on normal strengthen questions (as opposed to strengthen except), we normally eliminate answers like answer choice D that are sort of irrelevant/outside the scope (the conclusion in stimulus is about young children while answer choice D is about older children) since there is usually a better option?

Also, if we have something like answer choice D that is outside the scope like it is here, if there is no better strengthener, would it be ok to choose it on a regular Strengthen question? Thank you!
User avatar
 Beth Hayden
PowerScore Staff
  • PowerScore Staff
  • Posts: 123
  • Joined: Sep 04, 2021
|
#94427
Hi SDG,

On a strengthen question, the answer choice can be correct even if it only strengthens the conclusion a little bit.

Answer choice (D) does strengthen, and while it's about a slightly different group, it's not entirely out of scope. There is some connection generally between what happens with older and younger children, so it's not unreasonable to think that there might be a connection here. Just because the conclusion is about young children doesn't mean that you can't strengthen it with similar results from older children. It doesn't prove the conclusion, but if violent video games have a similar effect on older children, it makes it more likely that the conclusion is true.

If this was a normal strengthen question and not an except question, you wouldn't see (D) as a wrong answer with a "better" strengthener as the correct answer--the LSAT authors aren't going to do that to you!

Hope that helps,
Beth

Get the most out of your LSAT Prep Plus subscription.

Analyze and track your performance with our Testing and Analytics Package.